LSAT and Law School Admissions Forum

Get expert LSAT preparation and law school admissions advice from PowerScore Test Preparation.

User avatar
 Dave Killoran
PowerScore Staff
  • PowerScore Staff
  • Posts: 5853
  • Joined: Mar 25, 2011
|
#94177
Complete Question Explanation
(The complete setup for this game can be found here: viewtopic.php?f=385&p=94172)

The correct answer choice is (D)

Because F is a technician, only the 1-1-3 distribution allows F to be supervised by the president. Hence, answer choice (D) is correct.
 Paul Marsh
PowerScore Staff
  • PowerScore Staff
  • Posts: 290
  • Joined: Oct 15, 2019
|
#78030
This is a Local Must be True question. It gives us the new rule that F is supervised by the president. Two of our inferences that we made during Setup help us with this question. First, our inference that F must be a technician. Second, our inference that in the 1-2-2 distribution, each of the 2 managers supervises one of the 2 technicians (and therefore in that distribution it is not possible for the president to supervise a technician, since the technicians are all "claimed" by the managers). These two inferences can be combined with our new rule to say that we must be in the 1-1-3 distribution (since because F is a technician he can only be supervised by the president if we are in the 1-1-3 distribution).

In our 1-1-3 distribution, G can be either the manager or the president. So other than F, we can't say definitively place any variables. And since this is a Must be True question, we're only interested in what we can definitively say for sure; so we know that (A), (B), and (C) aren't going to be our answers. So we're left with (D) and (E), which ask about the numeric distribution of the game. Again, since we know that we're in the 1-1-3 distribution, (D) has to be our answer.

Get the most out of your LSAT Prep Plus subscription.

Analyze and track your performance with our Testing and Analytics Package.